www.vorwissen.de
Ein Projekt von vorhilfe.de
Das gesammelte Wissen der Vorhilfe
Hallo Gast!einloggen | registrieren ]
Startseite · Mitglieder · Teams · Forum · Wissen · Kurse · Impressum
Navigation
 Startseite...
 Neuerdings beta neu
 Forum...
 vorwissen...
 vorkurse...
 Werkzeuge...
 Nachhilfevermittlung beta...
 Online-Spiele beta
 Suchen
 Verein...
 Impressum
Das Projekt
Server und Internetanbindung werden durch Spenden finanziert.
Organisiert wird das Projekt von unserem Koordinatorenteam.
Hunderte Mitglieder helfen ehrenamtlich in unseren moderierten Foren.
Anbieter der Seite ist der gemeinnützige Verein "Vorhilfe.de e.V.".
Partnerseiten
Weitere Fächer:

Open Source FunktionenplotterFunkyPlot: Kostenloser und quelloffener Funktionenplotter für Linux und andere Betriebssysteme
Forum "Gewöhnliche Differentialgleichungen" - Anfangswertproblem
Anfangswertproblem < gewöhnliche < Differentialgl. < Analysis < Hochschule < Mathe < Vorhilfe
Ansicht: [ geschachtelt ] | ^ Forum "Gewöhnliche Differentialgleichungen"  | ^^ Alle Foren  | ^ Forenbaum  | Materialien

Anfangswertproblem: Korrektur
Status: (Frage) beantwortet Status 
Datum: 22:56 Di 22.07.2014
Autor: Phencyclidine

Aufgabe
Lösen sie folgendes AWP  mit y(1) = 0

y' = [mm] \wurzel[3]{x} [/mm] * e^ (-0,25y)





dy/dx  = [mm] \wurzel[3]{x} [/mm] * e^ (-0,25 y  )

Variablentrennung

[mm] \integral_{}^{}{e ^ (0,25y) dy } [/mm] = [mm] \integral_{}^{}{x^1/3 dx} [/mm]

4*e^(0,25y) + C1 = 3/4*x^(4/3) + C2

4*e^(0,25y) = 3/4*x*^4/3 + C

e^(0,25y) = [mm] 3/16*x^4/3 [/mm] + C/4

0,25y = ln (3/16x^(4/3)) + ln (C/4)

y = 4*ln(3/16x^(4/3)) + 4*ln(C/4)

Nun die Anfangsbedingung einsetzen.

0 = 4*ln(3/16*1*^(4/3)) +4 * ln(C/4)

0 = -6,69 + 4*ln(C/4)

6,69 = 4*ln(C/4)

1.67 = ln ( C/4)

e^(1,67) = C/4

5,3 = C/4

21,24 = C

Wollte fragen ob das soweit alles stimmt , oder irgentwo fehler auftreten. Es dient zur Übung einer Klausur aber ich habe leider keine Lösung.

Bitte um Hilfe!

Mit freundlichen Grüßen


        
Bezug
Anfangswertproblem: Antwort
Status: (Antwort) fertig Status 
Datum: 08:04 Mi 23.07.2014
Autor: Richie1401

Hallo,

das ist eigentlich überhaupt nicht lesbar. Kein Wunder also, dass noch niemand darauf geantwortet hat. Allgemein bitten wir dich die Formeln wirklich lesbar zu schreiben. Das ist auch positiv für dich, da du so schneller eine Antwort bekommst.

> Lösen sie folgendes AWP  mit y(1) = 0
>  
> y' = [mm]\wurzel[3]{x}[/mm] * e^ (-0,25y)
>  
>
>
>
> dy/dx  = [mm]\wurzel[3]{x}[/mm] * e^ (-0,25 y  )
>  
> Variablentrennung
>  
> [mm]\integral_{}^{}{e ^ (0,25y) dy }[/mm] = [mm]\integral_{}^{}{x^1/3 dx}[/mm]
>  
> 4*e^(0,25y) + C1 = 3/4*x^(4/3) + C2
>
> 4*e^(0,25y) = 3/4*x*^4/3 + C
>
> e^(0,25y) = [mm]3/16*x^4/3[/mm] + C/4
>  
> 0,25y = ln (3/16x^(4/3)) + ln (C/4)
>
> y = 4*ln(3/16x^(4/3)) + 4*ln(C/4)

Ist ok. Mit Logarithmengesetzen kann man das noch etwas kompakter zu

   [mm] y=4\ln\left(\frac{1}{4} \left(\frac{3 x^{4/3}}{4}+C\right)\right) [/mm]

>  
> Nun die Anfangsbedingung einsetzen.
>  
> 0 = 4*ln(3/16*1*^(4/3)) +4 * ln(C/4)
>  
> 0 = -6,69 + 4*ln(C/4)
>  
> 6,69 = 4*ln(C/4)
>
> 1.67 = ln ( C/4)
>  
> e^(1,67) = C/4
>  
> 5,3 = C/4
>
> 21,24 = C

Nutze keine Rundungen. Bitte bleib exakt bei den Werten.

>  
> Wollte fragen ob das soweit alles stimmt , oder irgentwo
> fehler auftreten. Es dient zur Übung einer Klausur aber
> ich habe leider keine Lösung.
>  
> Bitte um Hilfe!
>  
> Mit freundlichen Grüßen
>  


Bezug
        
Bezug
Anfangswertproblem: Antwort
Status: (Antwort) fertig Status 
Datum: 08:39 Mi 23.07.2014
Autor: fred97


> Lösen sie folgendes AWP  mit y(1) = 0
>  
> y' = [mm]\wurzel[3]{x}[/mm] * e^ (-0,25y)
>  
>
>
>
> dy/dx  = [mm]\wurzel[3]{x}[/mm] * e^ (-0,25 y  )
>  
> Variablentrennung
>  
> [mm]\integral_{}^{}{e ^ (0,25y) dy }[/mm] = [mm]\integral_{}^{}{x^1/3 dx}[/mm]
>  
> 4*e^(0,25y) + C1 = 3/4*x^(4/3) + C2
>
> 4*e^(0,25y) = 3/4*x*^4/3 + C
>
> e^(0,25y) = [mm]3/16*x^4/3[/mm] + C/4
>  
> 0,25y = ln (3/16x^(4/3)) + ln (C/4)
>
> y = 4*ln(3/16x^(4/3)) + 4*ln(C/4)
>  
> Nun die Anfangsbedingung einsetzen.
>  
> 0 = 4*ln(3/16*1*^(4/3)) +4 * ln(C/4)
>  
> 0 = -6,69 + 4*ln(C/4)
>  
> 6,69 = 4*ln(C/4)
>
> 1.67 = ln ( C/4)
>  
> e^(1,67) = C/4
>  
> 5,3 = C/4
>
> 21,24 = C

Ergänzend: bei der Berechnung von C hast Du Dich vertan.

FRED

>  
> Wollte fragen ob das soweit alles stimmt , oder irgentwo
> fehler auftreten. Es dient zur Übung einer Klausur aber
> ich habe leider keine Lösung.
>  
> Bitte um Hilfe!
>  
> Mit freundlichen Grüßen
>  


Bezug
Ansicht: [ geschachtelt ] | ^ Forum "Gewöhnliche Differentialgleichungen"  | ^^ Alle Foren  | ^ Forenbaum  | Materialien


^ Seitenanfang ^
www.vorwissen.de
[ Startseite | Mitglieder | Teams | Forum | Wissen | Kurse | Impressum ]